Find using Residue Theorem the following integral












0












$begingroup$


Find using Residue Theorem



$$int_{-1}^1 dfrac{dx}{(sqrt {1-x^2})(1+x^2)}$$



My try:



I took the contour $C=C_1+C_2 $ where $C_1$ is the upper half of the circle with center at $0$ and and radius $1$.



and $C_2$ is the line joining $-1$ to $1$.



I now consider the function in $Bbb C$ to be
$$int_C dfrac{dz}{(sqrt {1-z^2})(1+z^2)}$$



Over $C_2$ I get that $$int_{C_2} dfrac{dz}{(sqrt {1-z^2})(1+z^2)}$$=$$int_{-1}^1 dfrac{dx}{(sqrt {1-x^2})(1+x^2)}$$



But I dont know if I am doing it right or not because every singularity of the function lies on the boundary of $C$.



Also I cant figure out how to calculate $$int_{C_1} dfrac{dz}{(sqrt {1-z^2})(1+z^2)}$$



Any help from someone here?



Thanks for reading my post










share|cite|improve this question











$endgroup$












  • $begingroup$
    Either all the time there is lacking some parentheses in the denominator or else there is rather weird $;1;$ there...
    $endgroup$
    – DonAntonio
    Dec 4 '18 at 17:02












  • $begingroup$
    @DonAntonio;edited my question,can u have a look now
    $endgroup$
    – Join_PhD
    Dec 4 '18 at 17:07










  • $begingroup$
    Have you already chosen a branch for the square root function? There are two of them...But this isn't really that critical now: you cannot have a path from the point $;z=-1;$ to the point $;z=1;$ since on those points your function isn't defined. You can't also go through the point $;z=i;$ for the same reason.
    $endgroup$
    – DonAntonio
    Dec 4 '18 at 18:14


















0












$begingroup$


Find using Residue Theorem



$$int_{-1}^1 dfrac{dx}{(sqrt {1-x^2})(1+x^2)}$$



My try:



I took the contour $C=C_1+C_2 $ where $C_1$ is the upper half of the circle with center at $0$ and and radius $1$.



and $C_2$ is the line joining $-1$ to $1$.



I now consider the function in $Bbb C$ to be
$$int_C dfrac{dz}{(sqrt {1-z^2})(1+z^2)}$$



Over $C_2$ I get that $$int_{C_2} dfrac{dz}{(sqrt {1-z^2})(1+z^2)}$$=$$int_{-1}^1 dfrac{dx}{(sqrt {1-x^2})(1+x^2)}$$



But I dont know if I am doing it right or not because every singularity of the function lies on the boundary of $C$.



Also I cant figure out how to calculate $$int_{C_1} dfrac{dz}{(sqrt {1-z^2})(1+z^2)}$$



Any help from someone here?



Thanks for reading my post










share|cite|improve this question











$endgroup$












  • $begingroup$
    Either all the time there is lacking some parentheses in the denominator or else there is rather weird $;1;$ there...
    $endgroup$
    – DonAntonio
    Dec 4 '18 at 17:02












  • $begingroup$
    @DonAntonio;edited my question,can u have a look now
    $endgroup$
    – Join_PhD
    Dec 4 '18 at 17:07










  • $begingroup$
    Have you already chosen a branch for the square root function? There are two of them...But this isn't really that critical now: you cannot have a path from the point $;z=-1;$ to the point $;z=1;$ since on those points your function isn't defined. You can't also go through the point $;z=i;$ for the same reason.
    $endgroup$
    – DonAntonio
    Dec 4 '18 at 18:14
















0












0








0


0



$begingroup$


Find using Residue Theorem



$$int_{-1}^1 dfrac{dx}{(sqrt {1-x^2})(1+x^2)}$$



My try:



I took the contour $C=C_1+C_2 $ where $C_1$ is the upper half of the circle with center at $0$ and and radius $1$.



and $C_2$ is the line joining $-1$ to $1$.



I now consider the function in $Bbb C$ to be
$$int_C dfrac{dz}{(sqrt {1-z^2})(1+z^2)}$$



Over $C_2$ I get that $$int_{C_2} dfrac{dz}{(sqrt {1-z^2})(1+z^2)}$$=$$int_{-1}^1 dfrac{dx}{(sqrt {1-x^2})(1+x^2)}$$



But I dont know if I am doing it right or not because every singularity of the function lies on the boundary of $C$.



Also I cant figure out how to calculate $$int_{C_1} dfrac{dz}{(sqrt {1-z^2})(1+z^2)}$$



Any help from someone here?



Thanks for reading my post










share|cite|improve this question











$endgroup$




Find using Residue Theorem



$$int_{-1}^1 dfrac{dx}{(sqrt {1-x^2})(1+x^2)}$$



My try:



I took the contour $C=C_1+C_2 $ where $C_1$ is the upper half of the circle with center at $0$ and and radius $1$.



and $C_2$ is the line joining $-1$ to $1$.



I now consider the function in $Bbb C$ to be
$$int_C dfrac{dz}{(sqrt {1-z^2})(1+z^2)}$$



Over $C_2$ I get that $$int_{C_2} dfrac{dz}{(sqrt {1-z^2})(1+z^2)}$$=$$int_{-1}^1 dfrac{dx}{(sqrt {1-x^2})(1+x^2)}$$



But I dont know if I am doing it right or not because every singularity of the function lies on the boundary of $C$.



Also I cant figure out how to calculate $$int_{C_1} dfrac{dz}{(sqrt {1-z^2})(1+z^2)}$$



Any help from someone here?



Thanks for reading my post







complex-analysis residue-calculus






share|cite|improve this question















share|cite|improve this question













share|cite|improve this question




share|cite|improve this question








edited Dec 4 '18 at 17:06







Join_PhD

















asked Dec 4 '18 at 16:57









Join_PhDJoin_PhD

3898




3898












  • $begingroup$
    Either all the time there is lacking some parentheses in the denominator or else there is rather weird $;1;$ there...
    $endgroup$
    – DonAntonio
    Dec 4 '18 at 17:02












  • $begingroup$
    @DonAntonio;edited my question,can u have a look now
    $endgroup$
    – Join_PhD
    Dec 4 '18 at 17:07










  • $begingroup$
    Have you already chosen a branch for the square root function? There are two of them...But this isn't really that critical now: you cannot have a path from the point $;z=-1;$ to the point $;z=1;$ since on those points your function isn't defined. You can't also go through the point $;z=i;$ for the same reason.
    $endgroup$
    – DonAntonio
    Dec 4 '18 at 18:14




















  • $begingroup$
    Either all the time there is lacking some parentheses in the denominator or else there is rather weird $;1;$ there...
    $endgroup$
    – DonAntonio
    Dec 4 '18 at 17:02












  • $begingroup$
    @DonAntonio;edited my question,can u have a look now
    $endgroup$
    – Join_PhD
    Dec 4 '18 at 17:07










  • $begingroup$
    Have you already chosen a branch for the square root function? There are two of them...But this isn't really that critical now: you cannot have a path from the point $;z=-1;$ to the point $;z=1;$ since on those points your function isn't defined. You can't also go through the point $;z=i;$ for the same reason.
    $endgroup$
    – DonAntonio
    Dec 4 '18 at 18:14


















$begingroup$
Either all the time there is lacking some parentheses in the denominator or else there is rather weird $;1;$ there...
$endgroup$
– DonAntonio
Dec 4 '18 at 17:02






$begingroup$
Either all the time there is lacking some parentheses in the denominator or else there is rather weird $;1;$ there...
$endgroup$
– DonAntonio
Dec 4 '18 at 17:02














$begingroup$
@DonAntonio;edited my question,can u have a look now
$endgroup$
– Join_PhD
Dec 4 '18 at 17:07




$begingroup$
@DonAntonio;edited my question,can u have a look now
$endgroup$
– Join_PhD
Dec 4 '18 at 17:07












$begingroup$
Have you already chosen a branch for the square root function? There are two of them...But this isn't really that critical now: you cannot have a path from the point $;z=-1;$ to the point $;z=1;$ since on those points your function isn't defined. You can't also go through the point $;z=i;$ for the same reason.
$endgroup$
– DonAntonio
Dec 4 '18 at 18:14






$begingroup$
Have you already chosen a branch for the square root function? There are two of them...But this isn't really that critical now: you cannot have a path from the point $;z=-1;$ to the point $;z=1;$ since on those points your function isn't defined. You can't also go through the point $;z=i;$ for the same reason.
$endgroup$
– DonAntonio
Dec 4 '18 at 18:14












1 Answer
1






active

oldest

votes


















0












$begingroup$

The complex function has two branch points at $z=pm 1$ and a pole at $z=i$. Your contour contains all of these points, making it an inappropriate contour to use.



Let's pick a contour similar to the one seen in this example.



We pick the branch cut on $[-1,1]$ on the real line such that



begin{align}
1+z &= r_1e^{iphi_1}, & phi_1 in (-pi,pi] \
1-z &= r_2e^{iphi_2}, & phi_2 in (0,2pi)
end{align}



The contour consists of:





  • $C_1$: a line segment just above the branch cut going from $1+epsilon$ to $-1+epsilon$,


  • $C_2$: a left semicircle $z = -1 +epsilon e^{it}$, going counter-clockwise from $-1+epsilon$ to $-1-epsilon$


  • $C_3$: a line segment just below the branch cut, going from $-1-epsilon$ to $1-epsilon$


  • $C_4$: a right semicircle $z = 1 + epsilon e^{it}$, going counterclockwise from $1-epsilon$ to $1+epsilon$




In the limit of $epsilon to 0$, we have



$$ f(z)big|_{C_1} = frac{1}{(z^2+1)sqrt{|1+z|}e^{i0/2}sqrt{|1-z|}e^{i2pi/2}} = frac{-1}{(z^2+1)sqrt{|1+z|}sqrt{|1-z|}} $$



$$ f(z)big|_{C_3} = frac{1}{(z^2+1)sqrt{|1+z|}e^{-i0/2}sqrt{|1-z|}e^{i0/2}} = frac{1}{(z^2+1)sqrt{|1+z|}sqrt{|1-z|}} $$



$$ implies int_{C_1} f(z) dz + int_{C_3} f(z) dz = -int_1^{-1} f(x) dx + int_{-1}^1 f(x) dx = 2int_{-1}^1 f(x) dx $$





Next, we can prove the integral vanishes on the 2 semicircles. Using the ML inequality



$$ int_{C_2} f(z) dz le frac{L(C_2)}{|1+z^2|sqrt{|1-z|}sqrt{|1+z|}} le frac{pi epsilon}{2sqrt{2}sqrt{epsilon}} to 0 $$



Since $|z| ge 1$ and $|1-z| ge 2$



$$ int_{C_4} f(z) dz le frac{L(C_4)}{|1+z^2|sqrt{|1-z|}sqrt{|1+z|}} le frac{pi epsilon}{2sqrt{2}sqrt{epsilon}} to 0 $$



Since $|z| ge 1$ and $|1+z| ge 2$





Finally, use residues to finish the rest. You may also need to find the residue at infinity.






share|cite|improve this answer









$endgroup$













  • $begingroup$
    Thanks for an answer,I will need some time and patience to understand it
    $endgroup$
    – Join_PhD
    Dec 5 '18 at 10:09











Your Answer





StackExchange.ifUsing("editor", function () {
return StackExchange.using("mathjaxEditing", function () {
StackExchange.MarkdownEditor.creationCallbacks.add(function (editor, postfix) {
StackExchange.mathjaxEditing.prepareWmdForMathJax(editor, postfix, [["$", "$"], ["\\(","\\)"]]);
});
});
}, "mathjax-editing");

StackExchange.ready(function() {
var channelOptions = {
tags: "".split(" "),
id: "69"
};
initTagRenderer("".split(" "), "".split(" "), channelOptions);

StackExchange.using("externalEditor", function() {
// Have to fire editor after snippets, if snippets enabled
if (StackExchange.settings.snippets.snippetsEnabled) {
StackExchange.using("snippets", function() {
createEditor();
});
}
else {
createEditor();
}
});

function createEditor() {
StackExchange.prepareEditor({
heartbeatType: 'answer',
autoActivateHeartbeat: false,
convertImagesToLinks: true,
noModals: true,
showLowRepImageUploadWarning: true,
reputationToPostImages: 10,
bindNavPrevention: true,
postfix: "",
imageUploader: {
brandingHtml: "Powered by u003ca class="icon-imgur-white" href="https://imgur.com/"u003eu003c/au003e",
contentPolicyHtml: "User contributions licensed under u003ca href="https://creativecommons.org/licenses/by-sa/3.0/"u003ecc by-sa 3.0 with attribution requiredu003c/au003e u003ca href="https://stackoverflow.com/legal/content-policy"u003e(content policy)u003c/au003e",
allowUrls: true
},
noCode: true, onDemand: true,
discardSelector: ".discard-answer"
,immediatelyShowMarkdownHelp:true
});


}
});














draft saved

draft discarded


















StackExchange.ready(
function () {
StackExchange.openid.initPostLogin('.new-post-login', 'https%3a%2f%2fmath.stackexchange.com%2fquestions%2f3025823%2ffind-using-residue-theorem-the-following-integral%23new-answer', 'question_page');
}
);

Post as a guest















Required, but never shown

























1 Answer
1






active

oldest

votes








1 Answer
1






active

oldest

votes









active

oldest

votes






active

oldest

votes









0












$begingroup$

The complex function has two branch points at $z=pm 1$ and a pole at $z=i$. Your contour contains all of these points, making it an inappropriate contour to use.



Let's pick a contour similar to the one seen in this example.



We pick the branch cut on $[-1,1]$ on the real line such that



begin{align}
1+z &= r_1e^{iphi_1}, & phi_1 in (-pi,pi] \
1-z &= r_2e^{iphi_2}, & phi_2 in (0,2pi)
end{align}



The contour consists of:





  • $C_1$: a line segment just above the branch cut going from $1+epsilon$ to $-1+epsilon$,


  • $C_2$: a left semicircle $z = -1 +epsilon e^{it}$, going counter-clockwise from $-1+epsilon$ to $-1-epsilon$


  • $C_3$: a line segment just below the branch cut, going from $-1-epsilon$ to $1-epsilon$


  • $C_4$: a right semicircle $z = 1 + epsilon e^{it}$, going counterclockwise from $1-epsilon$ to $1+epsilon$




In the limit of $epsilon to 0$, we have



$$ f(z)big|_{C_1} = frac{1}{(z^2+1)sqrt{|1+z|}e^{i0/2}sqrt{|1-z|}e^{i2pi/2}} = frac{-1}{(z^2+1)sqrt{|1+z|}sqrt{|1-z|}} $$



$$ f(z)big|_{C_3} = frac{1}{(z^2+1)sqrt{|1+z|}e^{-i0/2}sqrt{|1-z|}e^{i0/2}} = frac{1}{(z^2+1)sqrt{|1+z|}sqrt{|1-z|}} $$



$$ implies int_{C_1} f(z) dz + int_{C_3} f(z) dz = -int_1^{-1} f(x) dx + int_{-1}^1 f(x) dx = 2int_{-1}^1 f(x) dx $$





Next, we can prove the integral vanishes on the 2 semicircles. Using the ML inequality



$$ int_{C_2} f(z) dz le frac{L(C_2)}{|1+z^2|sqrt{|1-z|}sqrt{|1+z|}} le frac{pi epsilon}{2sqrt{2}sqrt{epsilon}} to 0 $$



Since $|z| ge 1$ and $|1-z| ge 2$



$$ int_{C_4} f(z) dz le frac{L(C_4)}{|1+z^2|sqrt{|1-z|}sqrt{|1+z|}} le frac{pi epsilon}{2sqrt{2}sqrt{epsilon}} to 0 $$



Since $|z| ge 1$ and $|1+z| ge 2$





Finally, use residues to finish the rest. You may also need to find the residue at infinity.






share|cite|improve this answer









$endgroup$













  • $begingroup$
    Thanks for an answer,I will need some time and patience to understand it
    $endgroup$
    – Join_PhD
    Dec 5 '18 at 10:09
















0












$begingroup$

The complex function has two branch points at $z=pm 1$ and a pole at $z=i$. Your contour contains all of these points, making it an inappropriate contour to use.



Let's pick a contour similar to the one seen in this example.



We pick the branch cut on $[-1,1]$ on the real line such that



begin{align}
1+z &= r_1e^{iphi_1}, & phi_1 in (-pi,pi] \
1-z &= r_2e^{iphi_2}, & phi_2 in (0,2pi)
end{align}



The contour consists of:





  • $C_1$: a line segment just above the branch cut going from $1+epsilon$ to $-1+epsilon$,


  • $C_2$: a left semicircle $z = -1 +epsilon e^{it}$, going counter-clockwise from $-1+epsilon$ to $-1-epsilon$


  • $C_3$: a line segment just below the branch cut, going from $-1-epsilon$ to $1-epsilon$


  • $C_4$: a right semicircle $z = 1 + epsilon e^{it}$, going counterclockwise from $1-epsilon$ to $1+epsilon$




In the limit of $epsilon to 0$, we have



$$ f(z)big|_{C_1} = frac{1}{(z^2+1)sqrt{|1+z|}e^{i0/2}sqrt{|1-z|}e^{i2pi/2}} = frac{-1}{(z^2+1)sqrt{|1+z|}sqrt{|1-z|}} $$



$$ f(z)big|_{C_3} = frac{1}{(z^2+1)sqrt{|1+z|}e^{-i0/2}sqrt{|1-z|}e^{i0/2}} = frac{1}{(z^2+1)sqrt{|1+z|}sqrt{|1-z|}} $$



$$ implies int_{C_1} f(z) dz + int_{C_3} f(z) dz = -int_1^{-1} f(x) dx + int_{-1}^1 f(x) dx = 2int_{-1}^1 f(x) dx $$





Next, we can prove the integral vanishes on the 2 semicircles. Using the ML inequality



$$ int_{C_2} f(z) dz le frac{L(C_2)}{|1+z^2|sqrt{|1-z|}sqrt{|1+z|}} le frac{pi epsilon}{2sqrt{2}sqrt{epsilon}} to 0 $$



Since $|z| ge 1$ and $|1-z| ge 2$



$$ int_{C_4} f(z) dz le frac{L(C_4)}{|1+z^2|sqrt{|1-z|}sqrt{|1+z|}} le frac{pi epsilon}{2sqrt{2}sqrt{epsilon}} to 0 $$



Since $|z| ge 1$ and $|1+z| ge 2$





Finally, use residues to finish the rest. You may also need to find the residue at infinity.






share|cite|improve this answer









$endgroup$













  • $begingroup$
    Thanks for an answer,I will need some time and patience to understand it
    $endgroup$
    – Join_PhD
    Dec 5 '18 at 10:09














0












0








0





$begingroup$

The complex function has two branch points at $z=pm 1$ and a pole at $z=i$. Your contour contains all of these points, making it an inappropriate contour to use.



Let's pick a contour similar to the one seen in this example.



We pick the branch cut on $[-1,1]$ on the real line such that



begin{align}
1+z &= r_1e^{iphi_1}, & phi_1 in (-pi,pi] \
1-z &= r_2e^{iphi_2}, & phi_2 in (0,2pi)
end{align}



The contour consists of:





  • $C_1$: a line segment just above the branch cut going from $1+epsilon$ to $-1+epsilon$,


  • $C_2$: a left semicircle $z = -1 +epsilon e^{it}$, going counter-clockwise from $-1+epsilon$ to $-1-epsilon$


  • $C_3$: a line segment just below the branch cut, going from $-1-epsilon$ to $1-epsilon$


  • $C_4$: a right semicircle $z = 1 + epsilon e^{it}$, going counterclockwise from $1-epsilon$ to $1+epsilon$




In the limit of $epsilon to 0$, we have



$$ f(z)big|_{C_1} = frac{1}{(z^2+1)sqrt{|1+z|}e^{i0/2}sqrt{|1-z|}e^{i2pi/2}} = frac{-1}{(z^2+1)sqrt{|1+z|}sqrt{|1-z|}} $$



$$ f(z)big|_{C_3} = frac{1}{(z^2+1)sqrt{|1+z|}e^{-i0/2}sqrt{|1-z|}e^{i0/2}} = frac{1}{(z^2+1)sqrt{|1+z|}sqrt{|1-z|}} $$



$$ implies int_{C_1} f(z) dz + int_{C_3} f(z) dz = -int_1^{-1} f(x) dx + int_{-1}^1 f(x) dx = 2int_{-1}^1 f(x) dx $$





Next, we can prove the integral vanishes on the 2 semicircles. Using the ML inequality



$$ int_{C_2} f(z) dz le frac{L(C_2)}{|1+z^2|sqrt{|1-z|}sqrt{|1+z|}} le frac{pi epsilon}{2sqrt{2}sqrt{epsilon}} to 0 $$



Since $|z| ge 1$ and $|1-z| ge 2$



$$ int_{C_4} f(z) dz le frac{L(C_4)}{|1+z^2|sqrt{|1-z|}sqrt{|1+z|}} le frac{pi epsilon}{2sqrt{2}sqrt{epsilon}} to 0 $$



Since $|z| ge 1$ and $|1+z| ge 2$





Finally, use residues to finish the rest. You may also need to find the residue at infinity.






share|cite|improve this answer









$endgroup$



The complex function has two branch points at $z=pm 1$ and a pole at $z=i$. Your contour contains all of these points, making it an inappropriate contour to use.



Let's pick a contour similar to the one seen in this example.



We pick the branch cut on $[-1,1]$ on the real line such that



begin{align}
1+z &= r_1e^{iphi_1}, & phi_1 in (-pi,pi] \
1-z &= r_2e^{iphi_2}, & phi_2 in (0,2pi)
end{align}



The contour consists of:





  • $C_1$: a line segment just above the branch cut going from $1+epsilon$ to $-1+epsilon$,


  • $C_2$: a left semicircle $z = -1 +epsilon e^{it}$, going counter-clockwise from $-1+epsilon$ to $-1-epsilon$


  • $C_3$: a line segment just below the branch cut, going from $-1-epsilon$ to $1-epsilon$


  • $C_4$: a right semicircle $z = 1 + epsilon e^{it}$, going counterclockwise from $1-epsilon$ to $1+epsilon$




In the limit of $epsilon to 0$, we have



$$ f(z)big|_{C_1} = frac{1}{(z^2+1)sqrt{|1+z|}e^{i0/2}sqrt{|1-z|}e^{i2pi/2}} = frac{-1}{(z^2+1)sqrt{|1+z|}sqrt{|1-z|}} $$



$$ f(z)big|_{C_3} = frac{1}{(z^2+1)sqrt{|1+z|}e^{-i0/2}sqrt{|1-z|}e^{i0/2}} = frac{1}{(z^2+1)sqrt{|1+z|}sqrt{|1-z|}} $$



$$ implies int_{C_1} f(z) dz + int_{C_3} f(z) dz = -int_1^{-1} f(x) dx + int_{-1}^1 f(x) dx = 2int_{-1}^1 f(x) dx $$





Next, we can prove the integral vanishes on the 2 semicircles. Using the ML inequality



$$ int_{C_2} f(z) dz le frac{L(C_2)}{|1+z^2|sqrt{|1-z|}sqrt{|1+z|}} le frac{pi epsilon}{2sqrt{2}sqrt{epsilon}} to 0 $$



Since $|z| ge 1$ and $|1-z| ge 2$



$$ int_{C_4} f(z) dz le frac{L(C_4)}{|1+z^2|sqrt{|1-z|}sqrt{|1+z|}} le frac{pi epsilon}{2sqrt{2}sqrt{epsilon}} to 0 $$



Since $|z| ge 1$ and $|1+z| ge 2$





Finally, use residues to finish the rest. You may also need to find the residue at infinity.







share|cite|improve this answer












share|cite|improve this answer



share|cite|improve this answer










answered Dec 5 '18 at 10:07









DylanDylan

12.7k31026




12.7k31026












  • $begingroup$
    Thanks for an answer,I will need some time and patience to understand it
    $endgroup$
    – Join_PhD
    Dec 5 '18 at 10:09


















  • $begingroup$
    Thanks for an answer,I will need some time and patience to understand it
    $endgroup$
    – Join_PhD
    Dec 5 '18 at 10:09
















$begingroup$
Thanks for an answer,I will need some time and patience to understand it
$endgroup$
– Join_PhD
Dec 5 '18 at 10:09




$begingroup$
Thanks for an answer,I will need some time and patience to understand it
$endgroup$
– Join_PhD
Dec 5 '18 at 10:09


















draft saved

draft discarded




















































Thanks for contributing an answer to Mathematics Stack Exchange!


  • Please be sure to answer the question. Provide details and share your research!

But avoid



  • Asking for help, clarification, or responding to other answers.

  • Making statements based on opinion; back them up with references or personal experience.


Use MathJax to format equations. MathJax reference.


To learn more, see our tips on writing great answers.




draft saved


draft discarded














StackExchange.ready(
function () {
StackExchange.openid.initPostLogin('.new-post-login', 'https%3a%2f%2fmath.stackexchange.com%2fquestions%2f3025823%2ffind-using-residue-theorem-the-following-integral%23new-answer', 'question_page');
}
);

Post as a guest















Required, but never shown





















































Required, but never shown














Required, but never shown












Required, but never shown







Required, but never shown

































Required, but never shown














Required, but never shown












Required, but never shown







Required, but never shown







Popular posts from this blog

Plaza Victoria

In PowerPoint, is there a keyboard shortcut for bulleted / numbered list?

How to put 3 figures in Latex with 2 figures side by side and 1 below these side by side images but in...